What is the Big-Ω of the following function?












2












$begingroup$


For the following function:



$$
sum_{n=1}^{2n}x+x^2
$$



It is easy to see the (tightest) Big-Oh is $O(n^3)$, but I am not so sure about the Big-Omega. Here is my attempt:



$$
sum_{n=1}^{2n}x+x^2
$$

$$
geq sum_{n=1}^{2n}x^2
$$



But not sure how to continue. Also, does tightest Big-Ω always equal tightest Big-Oh?



EDIT: The way I worked out the Big-Oh is shown below:



$$
sum_{n=1}^{2n}x+x^2
$$

$$
leq sum_{n=1}^{2n}x^2+x^2
$$

$$
= sum_{n=1}^{2n}2x^2
$$

$$
leq 2n ( 2(2n)^2)
$$

$$
= 8n^3
$$



Therefore the Big-Oh is $O(n^3)$.










share|cite|improve this question











$endgroup$








  • 1




    $begingroup$
    Please show in the question how "to see the (tightest) Big-$O$ is $O(n^3)$".
    $endgroup$
    – Apass.Jack
    Jan 15 at 0:05










  • $begingroup$
    @Apass.Jack Updated the question.
    $endgroup$
    – TigerHix
    Jan 15 at 2:01
















2












$begingroup$


For the following function:



$$
sum_{n=1}^{2n}x+x^2
$$



It is easy to see the (tightest) Big-Oh is $O(n^3)$, but I am not so sure about the Big-Omega. Here is my attempt:



$$
sum_{n=1}^{2n}x+x^2
$$

$$
geq sum_{n=1}^{2n}x^2
$$



But not sure how to continue. Also, does tightest Big-Ω always equal tightest Big-Oh?



EDIT: The way I worked out the Big-Oh is shown below:



$$
sum_{n=1}^{2n}x+x^2
$$

$$
leq sum_{n=1}^{2n}x^2+x^2
$$

$$
= sum_{n=1}^{2n}2x^2
$$

$$
leq 2n ( 2(2n)^2)
$$

$$
= 8n^3
$$



Therefore the Big-Oh is $O(n^3)$.










share|cite|improve this question











$endgroup$








  • 1




    $begingroup$
    Please show in the question how "to see the (tightest) Big-$O$ is $O(n^3)$".
    $endgroup$
    – Apass.Jack
    Jan 15 at 0:05










  • $begingroup$
    @Apass.Jack Updated the question.
    $endgroup$
    – TigerHix
    Jan 15 at 2:01














2












2








2





$begingroup$


For the following function:



$$
sum_{n=1}^{2n}x+x^2
$$



It is easy to see the (tightest) Big-Oh is $O(n^3)$, but I am not so sure about the Big-Omega. Here is my attempt:



$$
sum_{n=1}^{2n}x+x^2
$$

$$
geq sum_{n=1}^{2n}x^2
$$



But not sure how to continue. Also, does tightest Big-Ω always equal tightest Big-Oh?



EDIT: The way I worked out the Big-Oh is shown below:



$$
sum_{n=1}^{2n}x+x^2
$$

$$
leq sum_{n=1}^{2n}x^2+x^2
$$

$$
= sum_{n=1}^{2n}2x^2
$$

$$
leq 2n ( 2(2n)^2)
$$

$$
= 8n^3
$$



Therefore the Big-Oh is $O(n^3)$.










share|cite|improve this question











$endgroup$




For the following function:



$$
sum_{n=1}^{2n}x+x^2
$$



It is easy to see the (tightest) Big-Oh is $O(n^3)$, but I am not so sure about the Big-Omega. Here is my attempt:



$$
sum_{n=1}^{2n}x+x^2
$$

$$
geq sum_{n=1}^{2n}x^2
$$



But not sure how to continue. Also, does tightest Big-Ω always equal tightest Big-Oh?



EDIT: The way I worked out the Big-Oh is shown below:



$$
sum_{n=1}^{2n}x+x^2
$$

$$
leq sum_{n=1}^{2n}x^2+x^2
$$

$$
= sum_{n=1}^{2n}2x^2
$$

$$
leq 2n ( 2(2n)^2)
$$

$$
= 8n^3
$$



Therefore the Big-Oh is $O(n^3)$.







asymptotics






share|cite|improve this question















share|cite|improve this question













share|cite|improve this question




share|cite|improve this question








edited Jan 15 at 7:44









Raphael

57.6k24140317




57.6k24140317










asked Jan 15 at 0:01









TigerHixTigerHix

134




134








  • 1




    $begingroup$
    Please show in the question how "to see the (tightest) Big-$O$ is $O(n^3)$".
    $endgroup$
    – Apass.Jack
    Jan 15 at 0:05










  • $begingroup$
    @Apass.Jack Updated the question.
    $endgroup$
    – TigerHix
    Jan 15 at 2:01














  • 1




    $begingroup$
    Please show in the question how "to see the (tightest) Big-$O$ is $O(n^3)$".
    $endgroup$
    – Apass.Jack
    Jan 15 at 0:05










  • $begingroup$
    @Apass.Jack Updated the question.
    $endgroup$
    – TigerHix
    Jan 15 at 2:01








1




1




$begingroup$
Please show in the question how "to see the (tightest) Big-$O$ is $O(n^3)$".
$endgroup$
– Apass.Jack
Jan 15 at 0:05




$begingroup$
Please show in the question how "to see the (tightest) Big-$O$ is $O(n^3)$".
$endgroup$
– Apass.Jack
Jan 15 at 0:05












$begingroup$
@Apass.Jack Updated the question.
$endgroup$
– TigerHix
Jan 15 at 2:01




$begingroup$
@Apass.Jack Updated the question.
$endgroup$
– TigerHix
Jan 15 at 2:01










1 Answer
1






active

oldest

votes


















3












$begingroup$

$$begin{align}
sum_{x=1}^{2n}(x+x^2) gt sum_{x=1}^{2n}x^2 gt sum_{x=n+1}^{2n}x^2gtsum_{x=n+1}^{2n}n^2= n^3
end{align}
$$



So the function is $Omega(n^3)$. You have shown it is $O(n^3)$. So the function is $Theta(n^3)$.



Although $n^3$, the asymptotic lower bound ignoring a constant factor is the same as the asymptotic upper bound ignoring a constant factor in the current case, it is not always true because some functions fluctuate. For example,
$$f(n)=begin{cases}1 quad text {when }ntext{ is odd,}\ n quad text{when }ntext{ is even.}end{cases}$$ $f(n)=O(n)$ and $f(n)=Omega(1)$. Both bounds are tightest.





Exercise. Show that $$sum_{x=1}^{m}(x+x^2)=frac{m(m+1)(m+2)}{3}$$






share|cite|improve this answer











$endgroup$









  • 1




    $begingroup$
    @TigerHix The fact that $O$ and $Omega$ can be different, btw, is why $Theta$ exists.
    $endgroup$
    – Draconis
    Jan 15 at 4:27








  • 1




    $begingroup$
    @Draconis That's a bit of a non-sequitur. There are situations where there may be a "nice" $Theta$-class (unlike for $f$ in the answer) but we can't prove tight bounds. Then, we need $O$ and $Omega$ to express that what we can prove.
    $endgroup$
    – Raphael
    Jan 15 at 7:46










  • $begingroup$
    @Raphael Sorry, what I'm saying is that if $O$ and $Omega$ were always the same, there'd be no reason to have $Theta$ as its own thing.
    $endgroup$
    – Draconis
    Jan 15 at 16:02










  • $begingroup$
    @Draconis I understood that, and I disagree. (If anything, it'd be a reason to have only $Theta$; arguably, $Theta$ is redundant.)
    $endgroup$
    – Raphael
    Jan 15 at 17:07











Your Answer





StackExchange.ifUsing("editor", function () {
return StackExchange.using("mathjaxEditing", function () {
StackExchange.MarkdownEditor.creationCallbacks.add(function (editor, postfix) {
StackExchange.mathjaxEditing.prepareWmdForMathJax(editor, postfix, [["$", "$"], ["\\(","\\)"]]);
});
});
}, "mathjax-editing");

StackExchange.ready(function() {
var channelOptions = {
tags: "".split(" "),
id: "419"
};
initTagRenderer("".split(" "), "".split(" "), channelOptions);

StackExchange.using("externalEditor", function() {
// Have to fire editor after snippets, if snippets enabled
if (StackExchange.settings.snippets.snippetsEnabled) {
StackExchange.using("snippets", function() {
createEditor();
});
}
else {
createEditor();
}
});

function createEditor() {
StackExchange.prepareEditor({
heartbeatType: 'answer',
autoActivateHeartbeat: false,
convertImagesToLinks: false,
noModals: true,
showLowRepImageUploadWarning: true,
reputationToPostImages: null,
bindNavPrevention: true,
postfix: "",
imageUploader: {
brandingHtml: "Powered by u003ca class="icon-imgur-white" href="https://imgur.com/"u003eu003c/au003e",
contentPolicyHtml: "User contributions licensed under u003ca href="https://creativecommons.org/licenses/by-sa/3.0/"u003ecc by-sa 3.0 with attribution requiredu003c/au003e u003ca href="https://stackoverflow.com/legal/content-policy"u003e(content policy)u003c/au003e",
allowUrls: true
},
onDemand: true,
discardSelector: ".discard-answer"
,immediatelyShowMarkdownHelp:true
});


}
});














draft saved

draft discarded


















StackExchange.ready(
function () {
StackExchange.openid.initPostLogin('.new-post-login', 'https%3a%2f%2fcs.stackexchange.com%2fquestions%2f102874%2fwhat-is-the-big-%25ce%25a9-of-the-following-function%23new-answer', 'question_page');
}
);

Post as a guest















Required, but never shown

























1 Answer
1






active

oldest

votes








1 Answer
1






active

oldest

votes









active

oldest

votes






active

oldest

votes









3












$begingroup$

$$begin{align}
sum_{x=1}^{2n}(x+x^2) gt sum_{x=1}^{2n}x^2 gt sum_{x=n+1}^{2n}x^2gtsum_{x=n+1}^{2n}n^2= n^3
end{align}
$$



So the function is $Omega(n^3)$. You have shown it is $O(n^3)$. So the function is $Theta(n^3)$.



Although $n^3$, the asymptotic lower bound ignoring a constant factor is the same as the asymptotic upper bound ignoring a constant factor in the current case, it is not always true because some functions fluctuate. For example,
$$f(n)=begin{cases}1 quad text {when }ntext{ is odd,}\ n quad text{when }ntext{ is even.}end{cases}$$ $f(n)=O(n)$ and $f(n)=Omega(1)$. Both bounds are tightest.





Exercise. Show that $$sum_{x=1}^{m}(x+x^2)=frac{m(m+1)(m+2)}{3}$$






share|cite|improve this answer











$endgroup$









  • 1




    $begingroup$
    @TigerHix The fact that $O$ and $Omega$ can be different, btw, is why $Theta$ exists.
    $endgroup$
    – Draconis
    Jan 15 at 4:27








  • 1




    $begingroup$
    @Draconis That's a bit of a non-sequitur. There are situations where there may be a "nice" $Theta$-class (unlike for $f$ in the answer) but we can't prove tight bounds. Then, we need $O$ and $Omega$ to express that what we can prove.
    $endgroup$
    – Raphael
    Jan 15 at 7:46










  • $begingroup$
    @Raphael Sorry, what I'm saying is that if $O$ and $Omega$ were always the same, there'd be no reason to have $Theta$ as its own thing.
    $endgroup$
    – Draconis
    Jan 15 at 16:02










  • $begingroup$
    @Draconis I understood that, and I disagree. (If anything, it'd be a reason to have only $Theta$; arguably, $Theta$ is redundant.)
    $endgroup$
    – Raphael
    Jan 15 at 17:07
















3












$begingroup$

$$begin{align}
sum_{x=1}^{2n}(x+x^2) gt sum_{x=1}^{2n}x^2 gt sum_{x=n+1}^{2n}x^2gtsum_{x=n+1}^{2n}n^2= n^3
end{align}
$$



So the function is $Omega(n^3)$. You have shown it is $O(n^3)$. So the function is $Theta(n^3)$.



Although $n^3$, the asymptotic lower bound ignoring a constant factor is the same as the asymptotic upper bound ignoring a constant factor in the current case, it is not always true because some functions fluctuate. For example,
$$f(n)=begin{cases}1 quad text {when }ntext{ is odd,}\ n quad text{when }ntext{ is even.}end{cases}$$ $f(n)=O(n)$ and $f(n)=Omega(1)$. Both bounds are tightest.





Exercise. Show that $$sum_{x=1}^{m}(x+x^2)=frac{m(m+1)(m+2)}{3}$$






share|cite|improve this answer











$endgroup$









  • 1




    $begingroup$
    @TigerHix The fact that $O$ and $Omega$ can be different, btw, is why $Theta$ exists.
    $endgroup$
    – Draconis
    Jan 15 at 4:27








  • 1




    $begingroup$
    @Draconis That's a bit of a non-sequitur. There are situations where there may be a "nice" $Theta$-class (unlike for $f$ in the answer) but we can't prove tight bounds. Then, we need $O$ and $Omega$ to express that what we can prove.
    $endgroup$
    – Raphael
    Jan 15 at 7:46










  • $begingroup$
    @Raphael Sorry, what I'm saying is that if $O$ and $Omega$ were always the same, there'd be no reason to have $Theta$ as its own thing.
    $endgroup$
    – Draconis
    Jan 15 at 16:02










  • $begingroup$
    @Draconis I understood that, and I disagree. (If anything, it'd be a reason to have only $Theta$; arguably, $Theta$ is redundant.)
    $endgroup$
    – Raphael
    Jan 15 at 17:07














3












3








3





$begingroup$

$$begin{align}
sum_{x=1}^{2n}(x+x^2) gt sum_{x=1}^{2n}x^2 gt sum_{x=n+1}^{2n}x^2gtsum_{x=n+1}^{2n}n^2= n^3
end{align}
$$



So the function is $Omega(n^3)$. You have shown it is $O(n^3)$. So the function is $Theta(n^3)$.



Although $n^3$, the asymptotic lower bound ignoring a constant factor is the same as the asymptotic upper bound ignoring a constant factor in the current case, it is not always true because some functions fluctuate. For example,
$$f(n)=begin{cases}1 quad text {when }ntext{ is odd,}\ n quad text{when }ntext{ is even.}end{cases}$$ $f(n)=O(n)$ and $f(n)=Omega(1)$. Both bounds are tightest.





Exercise. Show that $$sum_{x=1}^{m}(x+x^2)=frac{m(m+1)(m+2)}{3}$$






share|cite|improve this answer











$endgroup$



$$begin{align}
sum_{x=1}^{2n}(x+x^2) gt sum_{x=1}^{2n}x^2 gt sum_{x=n+1}^{2n}x^2gtsum_{x=n+1}^{2n}n^2= n^3
end{align}
$$



So the function is $Omega(n^3)$. You have shown it is $O(n^3)$. So the function is $Theta(n^3)$.



Although $n^3$, the asymptotic lower bound ignoring a constant factor is the same as the asymptotic upper bound ignoring a constant factor in the current case, it is not always true because some functions fluctuate. For example,
$$f(n)=begin{cases}1 quad text {when }ntext{ is odd,}\ n quad text{when }ntext{ is even.}end{cases}$$ $f(n)=O(n)$ and $f(n)=Omega(1)$. Both bounds are tightest.





Exercise. Show that $$sum_{x=1}^{m}(x+x^2)=frac{m(m+1)(m+2)}{3}$$







share|cite|improve this answer














share|cite|improve this answer



share|cite|improve this answer








edited Jan 15 at 3:41

























answered Jan 15 at 2:13









Apass.JackApass.Jack

10.8k1939




10.8k1939








  • 1




    $begingroup$
    @TigerHix The fact that $O$ and $Omega$ can be different, btw, is why $Theta$ exists.
    $endgroup$
    – Draconis
    Jan 15 at 4:27








  • 1




    $begingroup$
    @Draconis That's a bit of a non-sequitur. There are situations where there may be a "nice" $Theta$-class (unlike for $f$ in the answer) but we can't prove tight bounds. Then, we need $O$ and $Omega$ to express that what we can prove.
    $endgroup$
    – Raphael
    Jan 15 at 7:46










  • $begingroup$
    @Raphael Sorry, what I'm saying is that if $O$ and $Omega$ were always the same, there'd be no reason to have $Theta$ as its own thing.
    $endgroup$
    – Draconis
    Jan 15 at 16:02










  • $begingroup$
    @Draconis I understood that, and I disagree. (If anything, it'd be a reason to have only $Theta$; arguably, $Theta$ is redundant.)
    $endgroup$
    – Raphael
    Jan 15 at 17:07














  • 1




    $begingroup$
    @TigerHix The fact that $O$ and $Omega$ can be different, btw, is why $Theta$ exists.
    $endgroup$
    – Draconis
    Jan 15 at 4:27








  • 1




    $begingroup$
    @Draconis That's a bit of a non-sequitur. There are situations where there may be a "nice" $Theta$-class (unlike for $f$ in the answer) but we can't prove tight bounds. Then, we need $O$ and $Omega$ to express that what we can prove.
    $endgroup$
    – Raphael
    Jan 15 at 7:46










  • $begingroup$
    @Raphael Sorry, what I'm saying is that if $O$ and $Omega$ were always the same, there'd be no reason to have $Theta$ as its own thing.
    $endgroup$
    – Draconis
    Jan 15 at 16:02










  • $begingroup$
    @Draconis I understood that, and I disagree. (If anything, it'd be a reason to have only $Theta$; arguably, $Theta$ is redundant.)
    $endgroup$
    – Raphael
    Jan 15 at 17:07








1




1




$begingroup$
@TigerHix The fact that $O$ and $Omega$ can be different, btw, is why $Theta$ exists.
$endgroup$
– Draconis
Jan 15 at 4:27






$begingroup$
@TigerHix The fact that $O$ and $Omega$ can be different, btw, is why $Theta$ exists.
$endgroup$
– Draconis
Jan 15 at 4:27






1




1




$begingroup$
@Draconis That's a bit of a non-sequitur. There are situations where there may be a "nice" $Theta$-class (unlike for $f$ in the answer) but we can't prove tight bounds. Then, we need $O$ and $Omega$ to express that what we can prove.
$endgroup$
– Raphael
Jan 15 at 7:46




$begingroup$
@Draconis That's a bit of a non-sequitur. There are situations where there may be a "nice" $Theta$-class (unlike for $f$ in the answer) but we can't prove tight bounds. Then, we need $O$ and $Omega$ to express that what we can prove.
$endgroup$
– Raphael
Jan 15 at 7:46












$begingroup$
@Raphael Sorry, what I'm saying is that if $O$ and $Omega$ were always the same, there'd be no reason to have $Theta$ as its own thing.
$endgroup$
– Draconis
Jan 15 at 16:02




$begingroup$
@Raphael Sorry, what I'm saying is that if $O$ and $Omega$ were always the same, there'd be no reason to have $Theta$ as its own thing.
$endgroup$
– Draconis
Jan 15 at 16:02












$begingroup$
@Draconis I understood that, and I disagree. (If anything, it'd be a reason to have only $Theta$; arguably, $Theta$ is redundant.)
$endgroup$
– Raphael
Jan 15 at 17:07




$begingroup$
@Draconis I understood that, and I disagree. (If anything, it'd be a reason to have only $Theta$; arguably, $Theta$ is redundant.)
$endgroup$
– Raphael
Jan 15 at 17:07


















draft saved

draft discarded




















































Thanks for contributing an answer to Computer Science Stack Exchange!


  • Please be sure to answer the question. Provide details and share your research!

But avoid



  • Asking for help, clarification, or responding to other answers.

  • Making statements based on opinion; back them up with references or personal experience.


Use MathJax to format equations. MathJax reference.


To learn more, see our tips on writing great answers.




draft saved


draft discarded














StackExchange.ready(
function () {
StackExchange.openid.initPostLogin('.new-post-login', 'https%3a%2f%2fcs.stackexchange.com%2fquestions%2f102874%2fwhat-is-the-big-%25ce%25a9-of-the-following-function%23new-answer', 'question_page');
}
);

Post as a guest















Required, but never shown





















































Required, but never shown














Required, but never shown












Required, but never shown







Required, but never shown

































Required, but never shown














Required, but never shown












Required, but never shown







Required, but never shown







Popular posts from this blog

Can a sorcerer learn a 5th-level spell early by creating spell slots using the Font of Magic feature?

Does disintegrating a polymorphed enemy still kill it after the 2018 errata?

A Topological Invariant for $pi_3(U(n))$